A problem in Hoffman's Linear Algebra

Click For Summary
The discussion centers on a problem from Hoffman's Linear Algebra regarding diagonalizable linear operators and T-invariant subspaces. It asserts that if T is diagonalizable, every T-invariant subspace has a complementary T-invariant subspace, and vice versa. The participants suggest using the properties of eigenvectors and the self-adjoint nature of T to demonstrate this relationship. They emphasize that any T-invariant subspace can be spanned by eigenvectors, with its orthogonal complement also being T-invariant. The conversation also touches on the inversion of the proposition, seeking clarity on that aspect.
tghg
Messages
12
Reaction score
0
A problem in Hoffman's Linear Algebra.
Page 243

18. If T is a diagonalizable linear operator, then every T-invariant subspace has a complementary T-invariant subspace. And vice versa.

In fact, the answer lies on Pages 263~265,but I try not to use the conception T-admissible to prove this proposition.
Could someone help me out?
 
Last edited:
Physics news on Phys.org
There are a couple of different ways to do that:

The fact that T is "diagonalizable" means that there exist a basis for the vector space consisting entirely of eigenvectors of T (so that the matrix for T in that basis is diagonal). Using that, clearly any T-invariant subspace is spanned by some subset of those eigenvectors and it's orthogonal complement is spanned by the remaining eigenvectors- and so is T-invariant itself.

Or you could use the fact that, since T is diagonalizable, it is "self adjoint": for any vector u,v <Tu, v>= <u, Tv> where <u, v> is the inner product of u and v. That should make it easy to show that if a subspaced is T-invariant, then so is its orthogonal complement.
 
How about the Inversion of the proposition?
 
Question: A clock's minute hand has length 4 and its hour hand has length 3. What is the distance between the tips at the moment when it is increasing most rapidly?(Putnam Exam Question) Answer: Making assumption that both the hands moves at constant angular velocities, the answer is ## \sqrt{7} .## But don't you think this assumption is somewhat doubtful and wrong?

Similar threads

  • · Replies 10 ·
Replies
10
Views
2K
  • · Replies 1 ·
Replies
1
Views
1K
Replies
8
Views
2K
  • · Replies 3 ·
Replies
3
Views
2K
  • · Replies 3 ·
Replies
3
Views
2K
  • · Replies 4 ·
Replies
4
Views
2K
  • · Replies 6 ·
Replies
6
Views
4K
  • · Replies 8 ·
Replies
8
Views
2K
Replies
12
Views
9K
  • · Replies 3 ·
Replies
3
Views
3K